Đến nội dung

Kamii0909 nội dung

Có 155 mục bởi Kamii0909 (Tìm giới hạn từ 26-05-2020)



Sắp theo                Sắp xếp  

#665755 $QF$ tiếp xúc với $(CFM)$

Đã gửi bởi Kamii0909 on 24-12-2016 - 20:45 trong Hình học

nếu có $GC=GF$ ta suy ra $\widehat{CGM}=180^o-2\widehat{BFC}=90^o ,$ , từ $M$ kể tiếp tuyến $Mx$ của $(G)$ thì có hàng $M(xNBC=-1$ , suy ra $FQ$ tiếp xúc $(G)$ . Bây giờ ta đy chứng min $GF=GC$ , ta có$\frac{DF}{FC}=\frac{Sin\widehat{DBE}}{Sin\widehat{EBC}}=\frac{DE}{DB}.\frac{BC}{EC}=\frac{FO}{DG}$ suy ra tam giác $ODF$ đồng dạng $GCF$ suy ra tam giác $GFC$ cân tại $G$ suy ra $GF=GC$ suy ra dpcm

Bạn chỉ rõ cho mình tại sao $\frac{BC}{EC}.\frac{DE}{DB}=\frac{FO}{DG}$ được không? Bạn viết hơi tắt khúc này.



#663699 Chứng minh CD vuông góc OE

Đã gửi bởi Kamii0909 on 03-12-2016 - 14:48 trong Hình học

Còn 1 cách nữa suy nghĩ thêm đi :D

Cách khác.
Ta nhắc lại không chứng minh bổ đề quen thuộc sau.
Cho tứ giác nội tiếp $ABCD$ có 2 tiếp tuyến tại $B,D$ và $AC$ đồng quy. Khi đó 2 tiếp tuyến tại $A,C$ và $BD$ cũng đồng quy.(Tứ giác điều hòa)
Trở lại bài toán. Gọi $P$ là giao điểm $DC$ và $(O)$.
Khi đó theo bổ đề $EP$ là tiếp tuyến của $(O)$.
Từ đó dễ dàng dẫn đến đpcm.



#663696 Chứng minh CD vuông góc OE

Đã gửi bởi Kamii0909 on 03-12-2016 - 14:18 trong Hình học

Bài toán vẫn đúng trong trường hợp $A$ không là trung điểm $OD$.
Dễ dàng chứng minh được $\Delta OBD \sim \Delta ECB$
Nên $\frac{BO}{CE}=\frac{BD}{BC}=\frac{CO}{CE}$
Kết hợp với $\widehat{ECO}=\widehat{DBO}$ ta thu được $\Delta COE \sim \Delta BDC$ từ đó dễ dàng có đpcm.



#672386 Chứng minh $\sum \frac{1}{\sqrt{1+a^2...

Đã gửi bởi Kamii0909 on 22-02-2017 - 14:21 trong Bất đẳng thức - Cực trị

Có thể dùng dồn biến. Chú ý rằng với $ab\leq 1$ ta dễ dàng chỉ ra rằng
$$\frac{1}{\sqrt{1+a^2}}+\frac{1}{\sqrt{1+b^2}} \leq \frac{2}{\sqrt{ab+1}}$$. Thay $ab=\frac{1}{c}$ và xét hàm theo $c$.



#689907 $f(xy)=max\left \{ f(x+y), f(x).f(y) \right \}$

Đã gửi bởi Kamii0909 on 08-08-2017 - 15:25 trong Phương trình hàm

Bổ đề: Xét $a \geq 0$ và $g: \mathbb{R} \rightarrow \mathbb{R}$ thoả mãn $g(x)=x - \dfrac{a}{x}$ thì $g$ toàn ánh.(Không chứng minh) 

 

$P(x,y) : f(xy) = max \left \{ f(x+y),f(x)f(y) \right \}$

$P \left(x, \dfrac{-a}{x} \right) : f(-a)= max \left \{ f \left( x- \dfrac{a}{x} \right), f(x)f \left( \dfrac{-a}{x} \right) \right \}$

Tức là $f(-a) \geq f \left( x- \dfrac{a}{x} \right) (1)$
Từ đây do tính toàn ánh của $ x- \dfrac{a}{x}$ nên $f(-a) \geq f(x), \forall a \geq 0, x \in \mathbb{R}$

Hay $f(-y) \geq f(x), \forall y \geq 0, \forall x \in \mathbb{R} $

Từ đây $f(-y) \geq f(-x), \forall x,y \geq 0$ suy ra $f(x)=f(0)=C, \forall x \leq 0$ 

Ngoài ra, $f(x) \leq C, \forall x \in \mathbb{R}$

Xét $x \geq 0$

$P(-x,-1) : f(x)= max  \left \{ f(-x-1), f(-x)f(-1) \right \} = max \left \{ C, C^2  \right \}$

$\Rightarrow C \geq C^2 \Leftrightarrow 0 \ leq C \leq 1$

Mặt khác, với mọi $C \in [0,1]$ thì $C \geq C^2$ nên $f(x)=C, \forall x$




#694455 $f(x^2+f(xy))=xf(x+y)$

Đã gửi bởi Kamii0909 on 09-10-2017 - 17:23 trong Phương trình hàm

trước tiên ta nhận thấy pt có 1 ngh là f(x) đồng nhất bằng 0

ta thấy f(f(0))=0 thay y bởi f(0) trong pt đầu ta được f(x^2)=xf(x) suy ra f là hàm lẻ

suy ra luôn tồn tại số thực a thỏa f(a)=0

th1: a khác 0 lúc này thay x bởi a ta được f(x) là hàm hằng...... 

th2: suy ra chỉ có một giá trị là x=0 thỏa mãn f(x)=0 

thay x bởi -y ta được f(x^2)=x^2 mọi x thực 

lại có do tính lẻ của hàm f suy ra f(x)=x vs mọi x thực

Vậy.....

Làm đầy đủ chút được không bạn.
$P(x,f(0)):f(x^2+f(xf(0)))=xf(x+f(0))$
$P(a,y):f(a^2+f(ay))=af(y+a)$

Như bạn thấy cả 2 đẳng thức này chả thu được gì cả. 




#694174 $f(x^2+f(xy))=xf(x+y)$

Đã gửi bởi Kamii0909 on 04-10-2017 - 18:41 trong Phương trình hàm

Tìm tất cả $f:\mathbb{R} \rightarrow \mathbb{R}$ thoả mãn

$$f(x^2+f(xy))=xf(x+y), \forall x,y \in \mathbb{R}$$




#662810 Tìm n để trong $S_{n}$ không có số chính phương nào

Đã gửi bởi Kamii0909 on 23-11-2016 - 14:30 trong Số học

Bổ đề:Cho 2 số nguyên dương $a>b$
Nếu $\sqrt{a}-\sqrt{b} \geq 1$ thì $[a,b)$ có ít nhất 1 số chính phương.
*Lưu ý:Điều ngược lại là sai. Ví dụ $a=8,b=10$.
Như vậy ta phải có
$\sqrt{n}-\sqrt{\frac{n}{2}} <1$
Bằng biến đổi tương đương ta nhận được
$\sqrt{n} < \sqrt{2}(\sqrt{2}+1)$
Hay $n \leq 11$.
Thử trực tiếp có $n=9$ thỏa mãn.



#665696 $QF$ tiếp xúc với $(CFM)$

Đã gửi bởi Kamii0909 on 23-12-2016 - 23:54 trong Hình học

Cho hình vuông $ABCD$ nội tiếp $(O)$ và điểm $E$ trên cạnh $CD$. $AE$ cắt $BC$ tại $G$. $BE$ cắt $(O)$ tại $F$. Lấy $M \neq F$ trên $BE$ sao cho $GM=GF$. Gọi $N$ là trung điểm $BC$. $MN$ cắt $CD$ tại $Q$. Chứng minh rằng $QF$ tiếp xúc với $(CFM)$.



#661254 Tìm tất cả bộ các số nguyên tố sao cho tích của chúng bằng 10 lần tổng của ch...

Đã gửi bởi Kamii0909 on 09-11-2016 - 15:13 trong Số học

Làm gọn như sau
Dễ thấy có 2 số là 2 và 5.
Đặt số lớn nhất là $x$ thì
$VP \geq 2^{n-1} x$.
$VT \leq nx+7$
Từ đó $x(2^{n-1} -n) \leq 7$
Mà $x \geq 2 $ nên $2^{n-1} \leq 3+n$
Ta thấy điều này chỉ đúng với $n=1,2$
$n=1$ vô nghiệm nên $n=2$.



#658519 Cho các số thực x, y, z khác 1 và xyz=1. Chứng minh rằng $\sum...

Đã gửi bởi Kamii0909 on 20-10-2016 - 12:15 trong Bất đẳng thức và cực trị

Bạn tham khảo thêm cách khác 
Đặt $a= \frac{x}{x-1}\Leftrightarrow x= \frac{a}{a-1}$

$xyz=1 \Leftrightarrow abc= (a-1)(b-1)(c-1)\Leftrightarrow ab+bc+ac-a-b-c+1=0$

Ta có $(a+b+c-1)^{2}\geq 0\Leftrightarrow a^{2}+b^{2}+c^{2}\geq 1$




#657873 Đề Thi HSG Toán TP Hải Phòng ( bảng không chuyên ) năm 2016-2017

Đã gửi bởi Kamii0909 on 14-10-2016 - 22:40 trong Thi HSG cấp Tỉnh, Thành phố. Olympic 30-4. Đề thi và kiểm tra đội tuyển các cấp.

Câu 7 

Bằng biến đổi tương đương ta suy ra 

$P=\frac{3}{2}\left ( \sum x^{2} \right )+\frac{7}{2}$

Mà $\sum x^{2}\leq \left ( \sum x \right )^{2}\Leftrightarrow \sum xy\geq 0$

nên $P\leq 5$

Dấu bằng $\Leftrightarrow$ $\left ( x,y,z \right )=\left ( 0,0,1 \right )$




#659350 Tìm tất cả các cặp số nguyên dương (x;y) thỏa mãn: $x^6+x^3y=y^3+2y^2$

Đã gửi bởi Kamii0909 on 25-10-2016 - 22:34 trong Số học

Khúc đầu có thể làm đẹp hơn như sau 

Pt $(\frac{2x^3}{y}+1)^{2}=4y+9$

Từ đó đặt $k=\frac{2x^3}{y}$

Pt $k(k-2)(k+4)=8x^3$

nên $k=2a$

Pt $a(a-1)(a+2)=x^3$

Làm tương tự như trên sẽ ra  :icon6:  :icon6:

P/s:Mạng như ẹ  :mellow:  :mellow: Em gõ từ 9h mà mãi chưa được đi ngủ  :wacko:




#673878 Chứng minh $3(a+b+c)\geq \sqrt{8a^2+1}+\sqrt...

Đã gửi bởi Kamii0909 on 10-03-2017 - 17:40 trong Bất đẳng thức và cực trị

KMTTQ, $a \geq b \geq c$
Khi đó dễ cmr $a - \frac{1}{a} \geq b-\frac{1}{b} \geq c - \frac{1}{c}$
Và $3+ \sqrt{8+ \frac{1}{a^2}} \leq 3+ \sqrt{8+ \frac{1}{b^2}} \leq 3+\sqrt{8+\frac{1}{c^2}}$
Bđt cần cm tương đương với
$\sum \frac{a-\frac{1}{a}}{3+ \sqrt{8+\frac{1}{a^2}}} \geq 0$
Áp dụng bđt Cheybershev kết hợp điều kiện ta có đpcm.



#693855 $f:\mathbb{Z}_+\rightarrow \mathbb{Z}...

Đã gửi bởi Kamii0909 on 28-09-2017 - 15:38 trong Phương trình hàm

Tìm $f:\mathbb{Z}_+\rightarrow \mathbb{Z}_+$ thỏa mãn:

$f(\frac{f^2(n)}{n})=n$

Ta có $n|f^2(n)$ 

Từ tính chất này cho $n = \dfrac{f^2(n)}{n}$ thì $f^2(n)|n^3$

Đặt $n= p_{1}^{a_1} \cdot p_{2}^{a_2} ....p_{n}^{a_n}$

Trong đó $p_1,p_2,...,p_n$ nguyên tố còn $a_1,a_2,...,a_n \in \mathbb{N*}$

Do $f^2(n)|n^3$ nên $f(n)$ phải có dạng $p_{1}^{b_1} \cdot p_{2}^{b_2}...p_{n}^{b_n}$

Trong đó $b_1,b_2,...b_n \in \mathbb{N}$ và $2b_{i} \leq 3a_{i}$

Do đó $b_{i} \leq a_{i}$ hay $f(n)|n$

Từ đẳng thức này cho $n=\dfrac{f^2(n)}{n}$ thì $n^2|f^2(n)$

Do đó $f(n)=n,\forall n$.

Thử lại TM. 




#671492 $\sum (a^3-b^3)^2 \geq 3abc(a-b)(b-c)(c-a)$

Đã gửi bởi Kamii0909 on 13-02-2017 - 18:30 trong Bất đẳng thức - Cực trị

Bất đẳng thức vẫn đúng mà không cần điều kiện $a^5b+b^5c+c^5a=a^4b^2+b^4c^2+c^4a^2.$

Anh cho em tham khảo lời giải tổng quát? Và đặc biệt là hằng số tốt nhất. E có thử tìm nhưng nó khá là khó và chỉ dừng ở 6.
Em có thử với $a^5b+b^5c+c^5a \geq a^4b^2+b^4c^2+c^4a^2$ và cũng đã chứng minh được bất đẳng thức đúng tuy nhiên chưa làm được phần còn lại.



#658549 CMR: Năm điểm E; F; Q; D; P nằm trên 1 đường tròn.

Đã gửi bởi Kamii0909 on 20-10-2016 - 18:17 trong Hình học

Dễ có $\widehat{FPE}=\widehat{FQE}=60^{o}$ nên P,Q,F,E đồng viên 
$\widehat{MNP}+\widehat{NPQ}=180^{o}\Rightarrow \widehat{NPQ}=75^{o}\Rightarrow \widehat{QPF}=15^{o}\Rightarrow \widehat{QPF}+ \widehat{FQP}=90^{o}\Rightarrow \widehat{QFP}=90^{o}$




#669234 Chứng minh đường tròn đi qua 1 điểm cố định

Đã gửi bởi Kamii0909 on 21-01-2017 - 21:07 trong Hình học

Cho tứ giác $ABCD$ cố định và 1 điểm $P$ chuyển động trên đường $AC$. Giả sử đường tròn ngoại tiếp tam giác $APD$ cắt tia $AD$ tại $E$, đường tròn ngoại tiếp tam giác $APB$ cắt tai $AD$ tại $F$. Chứng minh rằng khi đó đường tròn ngoại tiếp tam giác $AEF$ luôn đi qua 1 điểm cố định.
[hide] Với cấu hình khá là đẹp thế này không biết liệu nó đã xuất hiện ở đâu chưa? [\hide]



#664004 Xác định vị trí của điểm $M$ để biểu thức: $P=\frac{...

Đã gửi bởi Kamii0909 on 06-12-2016 - 20:35 trong Hình học

Bài 1.
Xét TH $M$ thuộc cung nhỏ $AD$. Các TH còn lại chứng minh tương tự.
Lấy $G$ trên $AC$ sao cho $\widehat{BMC}=\widehat{AMG}$
Dễ có $\Delta BMC \sim \Delta AMG$ và $\Delta AMB \sim GMC$
Từ đó $\frac{AC}{ME}=\frac{AG}{ME} +\frac{GC}{ME}= \frac{BC}{MD}+ \frac{AB}{MF}$
Từ đó $P=\frac{2AC}{ME}$.
Dễ thấy $P$ không tồn tại GTNN.
Ở đây GTLN $P$ khi $MA=MC$.



#659947 C/m: Tâm ($MNH$) nằm trên $OH$.

Đã gửi bởi Kamii0909 on 30-10-2016 - 14:20 trong Hình học

APMO 2010 đây mà. Mình có đáp án mà nó bằng tiếng Anh nên ngại dịch lại quá 




#657053 Chứng minh rằng: $\sum \frac{1}{\sqrt{3+a}}\le...

Đã gửi bởi Kamii0909 on 07-10-2016 - 22:04 trong Bất đẳng thức và cực trị

$\sum \frac{1}{\sqrt{3+a}}\leq \sqrt{3\left ( \sum \frac{1}{a+3 } \right )}$

Ta sẽ chứng minh $\sum \frac{1}{3+a}\leq \frac{3}{4}$

Đổi biến $\left ( a,b,c \right )= \left ( \frac{x}{y},\frac{y}{z},\frac{z}{x} \right )$

Đpcm $\Leftrightarrow \sum \frac{y}{x+3y}\leq \frac{3}{4}\Leftrightarrow \sum \frac{3y}{x+3y}\leq \frac{9}{4}\Leftrightarrow \sum \frac{x}{x+3y}\geq \frac{3}{4}$

Điều này luôn đúng do $\sum \frac{x}{x+3y}= \sum \frac{x^{2}}{x^{2}+3xy}\geq \frac{\left ( \sum x \right )^{2}}{\sum x^{2}+3\sum xy}= \frac{\left ( \sum x \right )^{2}}{\left ( \sum x \right )^{2}+\sum xy}\geq \frac{\left ( \sum x \right )^{2}}{\left ( \sum x \right )^{2}+\frac{1}{3}\left ( \sum x \right )^{2}}=\frac{3}{4}$




#659818 Tìm các số nguyên dương sao cho $\frac{a^2+b}{b^2-a}$ và...

Đã gửi bởi Kamii0909 on 29-10-2016 - 16:25 trong Số học

Viết  lại cách trên dễ hiểu hơn. Ta viết lại điều kiện $\left\{\begin{matrix} b^2-a|a^2+b\\ a^2-b|b^2+a \end{matrix}\right.$

Từ đó $\left\{\begin{matrix} a^2+b \geq b^2-a\\ b^2+a \geq a^2-b \end{matrix}\right.\Leftrightarrow \left\{\begin{matrix} (a-b+1)(a+b)\geq 0\\ (b-a+1)(a+b) \geq 0 \end{matrix}\right.$

Do $a+b >0$ nên $\left\{\begin{matrix} a-b+1 \leq 0\\ b-a+1 \leq 0 \end{matrix}\right.\Rightarrow (a-b+1)(b-a+1) \leq 0 \Leftrightarrow (a-b)^2 \leq 1$

Giả sử $a \geq b$ thì $a=b+1$ hoặc $a=b$

Nếu $a=b+1$ thì $a^2-a-1|a^2+3a+1$ hay $a^2-a-1|4a+2$

Từ đó $a^2-a-1 \leq 4a +2$ suy ra $a \leq 5$. Thay lại tìm $b$ 

Nếu $a=b$ thì $a^2-a|a^2+a$ hay $a^2-a|2a$ nên $a(a-3) \leq 0$ 

Vậy $a \leq 3 $ Thay lại ta tìm được nốt. 




#669214 Tìm min $P=\frac{1}{x^{2}+1}+\fr...

Đã gửi bởi Kamii0909 on 21-01-2017 - 20:03 trong Bất đẳng thức - Cực trị

Bài toán nên là $x,y,z$ không âm thì hay hơn( khi đó dấu bằng xảy ra thêm tại $(x,y,z) \sim (0,\sqrt{3},\sqrt{3})$)

Ta sẽ chứng minh rằng $\sum \dfrac{1}{x^2+1} \geq \frac{3}{2}.$

Nhân lên và biến đổi $p,q,r,$ ta đi chứng minh $p^2 +pr +r(p-3r) \geq 12.$

Do $a+b+c \geq 3 \sqrt[3]{abc} \geq 3abc \Rightarrow p \geq 3r,$ nên ta quy về chứng minh $p^2+pr \geq 12.$

Nếu $p^2 \geq 12$ thì bất đẳng thức hiển nhiên đúng. Giả sử $9 \leq p^2 \leq 12.$

Áp dụng bất đẳng thức Schur bậc 3 $r \geq \frac{p(12-p^2)}{9},$ điều phải chứng minh trở thành  $(p^2-9)(12-p^2) \geq 0$ :đúng.




#666871 $\sum \dfrac{abc+b+c-a}{a^2+1} \geq...

Đã gửi bởi Kamii0909 on 03-01-2017 - 22:23 trong Bất đẳng thức và cực trị

Với $a,b,c \geq 0$ chứng minh rằng
$\dfrac{abc+b+c-a}{a^2+1}+\dfrac{abc+c+a-b}{b^2+1} +\dfrac{abc+a+b-c}{c^2+1} \geq a+b+c$



#671421 $\sum (a^3-b^3)^2 \geq 3abc(a-b)(b-c)(c-a)$

Đã gửi bởi Kamii0909 on 13-02-2017 - 02:50 trong Bất đẳng thức - Cực trị

Cho $a,b,c$ là các số thực thỏa mãn $a^5b+b^5c+c^5a=a^4b^2+b^4c^2+c^4a^2$
Chứng minh rằng
$(a^3-b^3)^2+(b^3-c^3)^2+(c^3-a^3)^2 \geq 6abc(a-b)(b-c)(c-a)$